S&Jv8,p27.01: Add missing period to the question
[course.git] / latex / problems / Serway_and_Jewett_8 / problem23.17.tex
1 \begin{problem*}{23.17}
2 A point charge $+2Q$ is at the origin and a point charge $-Q$ is
3 located along the $x$ axis at $x=d$ as in Figure~P23.17.  Find a
4 symbolic expression for the net force on a third point charge $+Q$
5 located along the $y$ axis at $y=d$.
6 \begin{center}
7 \begin{asy}
8 import Mechanics;
9 import ElectroMag;
10
11 real u = 1.5cm;
12
13 Charge a = pCharge((0,0), 2, L=Label("$2Q$", align=S));
14 Charge b = nCharge((u,0), -1, L=Label("$-Q$", align=S));
15 Charge c = pCharge((0,u), 1, L=Label("$Q$", align=S));
16 Distance dab = Distance(
17     a.center(), b.center(), offset=0.5u, scale=u, L="$d$");
18 Distance dac = Distance(
19     a.center(), c.center(), offset=-0.5u, scale=u, L="$d$");
20 draw_ijhat((0,0));
21 a.draw(); b.draw(); c.draw(); dab.draw(); dac.draw();
22 \end{asy}
23 \end{center}
24 \end{problem*}
25
26 \begin{solution}
27 Summing the forces from the other two charges
28 \begin{equation}
29   \vect{F} = \vect{F}_0 + \vect{F}_x
30     = k\frac{Q\cdot2Q}{d^2}\jhat
31       + k\frac{Q\cdot(-Q)}{d^2+d^2}\frac{-\ihat+\jhat}{\sqrt{2}}
32     = k\frac{Q^2}{d^2}\p({ 2\jhat - \frac{1}{2\sqrt{2}}(-\ihat+\jhat) })
33     = \ans{k\frac{Q^2}{d^2}\p[{ \frac{\ihat}{2\sqrt{2}}
34                                + \p({2-\frac{1}{2\sqrt{2}}})\jhat }] }
35 \end{equation}
36 \end{solution}